Disguised Inequality

Algebra Level 4

( a + b + c ) ( a 2 + b 2 + c 2 ) + 3 a b c N ( a + b ) ( a + c ) ( b + c ) (a+b+c)(a^2+b^2+c^2)+3abc\geq N(a+b)(a+c)(b+c)

If a , b , c a,b,c and N N are positive reals such that the maximum value of N N which satisfies the inequality above can be expressed in the form m n \frac{m}{n} , where m m and n n are coprime positive integers, find the value of m + n m+n .


The answer is 5.

This section requires Javascript.
You are seeing this because something didn't load right. We suggest you, (a) try refreshing the page, (b) enabling javascript if it is disabled on your browser and, finally, (c) loading the non-javascript version of this page . We're sorry about the hassle.

3 solutions

Calvin Lin Staff
Feb 20, 2016

We suspect that equality occurs when a = b = c a = b = c , which yield a value of N = 3 2 N = \frac{3}{2} . Thus, we want to show that

2 [ ( a + b + c ) ( a 2 + b 2 + c 2 ) + 3 a b c ] 3 ( a + b ) ( b + c ) ( c + a ) . 2 [ (a+b+c) ( a^2 + b^2 + c^2 ) + 3abc ] \geq 3 (a+b)(b+c)(c+a).

Expanding both sides and subtracting terms, this is equivalent to

2 a 3 + 2 b 3 + 2 c 3 a 2 b + b 2 c + c 2 a + a 2 c + b 2 a + c 2 b , 2a^3 + 2b^3 + 2c^3 \geq a^2b + b^2 c + c^2 a + a^2 c + b^2 a + c^2b,

which is obviously true.

How is the last inequality obvious (it's not obvious to me)?

Deeparaj Bhat - 5 years, 3 months ago

Log in to reply

Zk Lin
Feb 14, 2016

Note that

L.H.S. = a 3 + b 3 + c 3 + 3 a b c + s y m a 2 b =a^{3}+b^{3}+c^{3}+3abc +\displaystyle \sum_{sym} a^{2}b

2 s y m a 2 b \geq 2 \displaystyle \sum_{sym} a^{2}b by Schur's inequality

12 a b c \geq 12abc by AM-GM inequality.

Equality is attained when a = b = c a=b=c .

R.H.S. = 2 N a b c + N s y m a 2 b =2Nabc+N\displaystyle \sum_{sym} a^{2}b

2 N a b c + 6 N a b c \geq 2Nabc+ 6Nabc by AM-GM inequality

= 8 N a b c =8Nabc

Equality is attained when a = b = c a=b=c .

Since L.H.S. \geq R.H.S. 8 N a b c \geq 8Nabc ,

we have L.H.S. 8 N a b c \geq 8Nabc .

However, we have already proven that the minimum of L.H.S. is attained at 12 a b c 12abc .

Therefore 8 N = 12 8N=12 , which simplifies to N = 3 2 N=\frac{3}{2} .

This gives m = 3 , n = 2 m=3,n=2 , hence the answer 3 + 2 = 5 3+2=\boxed{5} .

Moderator note:

This solution is non-sensical. Do you see why?

This solution is non-sensical. Do you see why?

Calvin Lin Staff - 5 years, 3 months ago

Log in to reply

@Calvin Lin I am not quite sure, but I think it has to do with

Since L.H.S. \geq R.H.S. 8 N a b c \geq 8Nabc ,

we have L.H.S. 8 N a b c \geq 8Nabc .

Can you explain why this approach is wrong?

ZK LIn - 5 years, 3 months ago

Log in to reply

Depending on your interpretation / thoughts as you were writing it up:

  • Even if it's true that L H S 8 a b c LHS \geq 8 abc , how does that imply that L H S 3 2 ( a + b ) ( b + c ) ( c + a ) LHS \geq \frac{3}{2} (a+b)(b+c)(c+a) ? All that we have is 3 2 ( a + b ) ( b + c ) ( c + a ) 8 a b c \frac{3}{2} (a+b)(b+c)(c+a) \geq 8 abc .
  • In essence, we want L H S ( a + b ) ( b + c ) ( c + a ) N \frac{LHS}{ (a+b)(b+c)(c+a) } \geq N . To continue your approach (where we treated the terms independently, we have to minimize LHS and maximize ( a + b ) ( b + c ) ( c + a ) (a+b)(b+c)(c+a) .
  • It is useful to list inequalities as "Want to show" or "It is equaivalent to show that" VS "We know that". It can be easy to mix these up when we have a long list of statements written down.

Calvin Lin Staff - 5 years, 3 months ago

Log in to reply

@Calvin Lin @Calvin Lin In order to try to understand what has gone wrong, I go over my thought process while solving this question and try to reconstruct the whole argument backwards, starting from the fact that L.H.S. 12 a b c \geq 12abc (We know this is true from Schur's + AM-GM). Therefore, if L.H.S. 8 N a b c \geq 8Nabc , then such greatest N N is 12 8 = 3 2 \frac{12}{8}=\frac{3}{2} .

From L.H.S. 8 N a b c \geq 8Nabc , we have L.H.S \geq R.H.S. 8 N a b c \geq 8Nabc . The first part of the inequality is given in the question while R.H.S. 8 N a b c \geq 8Nabc is separately proven above .

Therefore, we have L.H.S. \geq R.H.S , or

( a + b + c ) ( a 2 + b 2 + c 2 ) + 3 a b c N ( a + b ) ( a + c ) ( b + c ) (a+b+c)(a^2+b^2+c^2)+3abc\geq N(a+b)(a+c)(b+c)

substitute N = 3 2 N=\frac{3}{2} , the inequality should in theory, hold, and we note that such N N is the maximum permitted.


I think the bolded part is fallacious. I committed a logical fallacy while assuming L.H.S \geq R.H.S. 8 N a b c \geq 8Nabc implies L.H.S. \geq R.H.S for all N N in the backward proof when in actual fact, the sign of the inequality might flip depending on the value of N N . In other words, I cannot assume that the inequality holds, find the maximum N = 3 2 N=\frac{3}{2} and claim that I have solved the problem.

Similarly, in my original proof (forward proof), I cannot infer from L.H.S \geq R.H.S. \geq 8 N a b c 8Nabc that L.H.S. 8 N a b c \geq 8Nabc . Who knows for certain values of N N , the inequality flips to R.H.S. 8 N a b c \geq 8Nabc \geq L.H.S.

Now that I have noticed this, this fallacy is indeed quite dangerous. While typing up the solution, it didn't even cross my mind that I have assumed things I shouldn't.

What do you think about it? Do I correctly identify the flaw in my proof?

ZK LIn - 5 years, 3 months ago

Log in to reply

@Zk Lin Right, we have shown that L H S 12 a b c LHS \geq 12 abc and R H S × N 8 a b c × N RHS \times N \geq 8abc \times N . This however does not imply that L H S 3 2 R H S LHS \geq \frac{3}{2} RHS , since we need to bound the RHS from above, instead of from below.

This is a pretty common mistake made when manipulating such "fractional" inequalities, and not being careful about how we're multiplying / comparing them. That's one reason why Titu's lemma is so powerful, because it helps you make the correct division step.

Calvin Lin Staff - 5 years, 3 months ago

Sorry, this is incorrect. \color{#D61F06}{\text{Sorry, this is incorrect.}}

We note that:

( a + b ) ( b + c ) ( c + a ) = ( a + b + c ) ( a b + b c + c a ) a b c (a+b)(b+c)(c+a) = (a+b+c)(ab+bc+ca)-abc

Using Cauchy-Schwarz inequality we have:

a + b + c 3 ( a b c ) 1 3 and a b + b c + c a 3 ( a b c ) 2 3 a+b+c \ge 3(abc)^{\frac{1}{3}}\quad \text{and} \quad ab+bc+ca \ge 3(abc)^{\frac{2}{3}}

( a + b + c ) ( a b + b c + c a ) 9 a b c ( a + b ) ( b + c ) ( c + a ) 8 a b c \Rightarrow (a+b+c)(ab+bc+ca) \ge 9abc\quad \Rightarrow (a+b)(b+c)(c+a) \ge 8abc

We also note that:

( a b + b c + c a ) 2 ( a 2 + b 2 + c 2 ) ( a 2 + b 2 + c 2 ) a 2 + b 2 + c 2 a b + b c + c a ( a + b + c ) ( a 2 + b 2 + c 2 ) ( a + b + c ) ( a b + b c + c a ) ( a + b + c ) ( a 2 + b 2 + c 2 ) ( a + b ) ( b + c ) ( c + a ) + a b c ( a + b + c ) ( a 2 + b 2 + c 2 ) + 3 a b c ( a + b ) ( b + c ) ( c + a ) + 4 a b c ( a + b + c ) ( a 2 + b 2 + c 2 ) + 3 a b c ( a + b ) ( b + c ) ( c + a ) + 4 8 ( a + b ) ( b + c ) ( c + a ) ( a + b + c ) ( a 2 + b 2 + c 2 ) + 3 a b c 3 2 ( a + b ) ( b + c ) ( c + a ) \begin{aligned} (ab+bc+ca)^2 & \le (a^2+b^2+c^2)(a^2+b^2+c^2) \\ a^2+b^2+c^2 & \ge ab+bc+ca \\ (a+b+c)(a^2+b^2+c^2) & \ge (a+b+c)(ab+bc+ca) \\(a+b+c)(a^2+b^2+c^2) & \ge (a+b)(b+c)(c+a) + abc \\ (a+b+c)(a^2+b^2+c^2) + 3abc & \ge (a+b)(b+c)(c+a) + 4abc \\ (a+b+c)(a^2+b^2+c^2) + 3abc & \ge (a+b)(b+c)(c+a) + \frac {4}{8} (a+b)(b+c)(c+a) \\ (a+b+c)(a^2+b^2+c^2) + 3abc & \ge \frac {3}{2} (a+b)(b+c)(c+a) \end{aligned}

m = 3 \Rightarrow m = 3 , n = 2 n=2 and m + n = 5 m+n = \boxed{5}

I don't understand how does the 4th last line imply the 3rd last line. isn't 1/8(a+b)(b+c)(c+a) more than abc?

Wuu Yyiizzhhoouu - 5 years, 4 months ago

Log in to reply

You are right. Let me redo the problem.

Chew-Seong Cheong - 5 years, 4 months ago

Log in to reply

thank you so much!

Wuu Yyiizzhhoouu - 5 years, 4 months ago

You should also write the equality cases as each may have been different from one another.

Reineir Duran - 5 years, 4 months ago

0 pending reports

×

Problem Loading...

Note Loading...

Set Loading...